Differentiating Parametric Functions

Click For Summary
To find dy/dx for the parametric equations x = 2cos(t) and y = 2sin(t), the derivatives dy/dt and dx/dt are calculated as dy/dt = 2cos(t) and dx/dt = -2sin(t). This leads to dy/dx = (dy/dt) / (dx/dt) = -cot(t). The user initially struggles with a discrepancy between their result and the book's answer, which is not expressed in trigonometric terms. Ultimately, they realize that squaring the x and y functions and adding them eliminates the parameter, providing a different approach to the problem.
danago
Gold Member
Messages
1,118
Reaction score
4

Given:
<br /> \begin{array}{l}<br /> x = 2\cos t \\ <br /> y = 2\sin t \\ <br /> \end{array}<br />

find <br /> \frac{{dy}}{{dx}}<br />



I started by finding dy/dt and dx/dt

<br /> \begin{array}{l}<br /> \frac{{dy}}{{dt}} = 2\cos t \\ <br /> \frac{{dx}}{{dt}} = - 2\sin t \\ <br /> \end{array}<br />

Now, dy/dx = (dy/dt) / (dx/dt)

so:

<br /> \frac{{dy}}{{dx}} = \frac{{2\cos t}}{{ - 2\sin t}} = - \cot t<br />

<br /> \begin{array}{l}<br /> x = 2\cos t \\ <br /> \therefore t = \cos ^{ - 1} \frac{x}{2} \\ <br /> \end{array}<br />

Substituting that into my dy/dx which is currently in terms of t:

<br /> \frac{{dy}}{{dx}} = - \cot t = - \cot \cos ^{ - 1} \frac{x}{2}<br />

Now, my book gives a completely different answer, which isn't even in terms of any trigonometric function. Where have i gone wrong?

Thanks,
Dan.
 
Physics news on Phys.org
Do you know that sin2t + cos2t = 1? ;)
 
neutrino said:
Do you know that sin2t + cos2t = 1? ;)

Yep i do. Not quite seeing where it comes into it though :cry:

EDIT: ahhh now I've got it. Square my x and y functions, and then add them, which eliminates the parameter :) Thanks for that.
 
Last edited:
You could also continue on your original course. cot(cos^(-1)(x/2)) can be written without use of trig functions. Draw a right triangle containing an angle cos^(-1)(x/2).
 
Question: A clock's minute hand has length 4 and its hour hand has length 3. What is the distance between the tips at the moment when it is increasing most rapidly?(Putnam Exam Question) Answer: Making assumption that both the hands moves at constant angular velocities, the answer is ## \sqrt{7} .## But don't you think this assumption is somewhat doubtful and wrong?

Similar threads

  • · Replies 8 ·
Replies
8
Views
1K
Replies
3
Views
2K
  • · Replies 20 ·
Replies
20
Views
2K
Replies
3
Views
2K
Replies
6
Views
1K
Replies
12
Views
2K
  • · Replies 1 ·
Replies
1
Views
2K
  • · Replies 3 ·
Replies
3
Views
2K
  • · Replies 5 ·
Replies
5
Views
2K
  • · Replies 15 ·
Replies
15
Views
2K